You are on page 1of 47

Potanin Bogdan Stanislavovich

Microeconomics 2015 Friedman exercices

1 Seminars
1. Alice has utility function u A = x 1 x2 . Donna has utility function uD = 10+x1 1 x2 Do
they have the same preferences?
Solution Utility functions are equal, if their MRS are the same.
A x2 D (10+x1 x2 )2 x2 x2 A D
M RS1,2 = x1
, M RS1,2 = (10+x1 x2 )2 x1
= x1
=> M RS1,2 = M RS1,2 => functions are
equal.
2. Suppose, that consumer has preferences that satisfy the four axioms discussed in
class. If in addition M RSx,y = 0 when y=0, then consumer always purchases positive
amount of y at any px , py > 0. True or false?
Solution
If solution is corner and only y
consumed, then M RSx,y
px
py
. Samely, if only x
px px
consumed M RSx,y . Here M RSx,y (x , 0) = 0 then only x consumption is
py py
impossible thus some of y is neccessary purchased. So the statment is true.
3. George is a consumer whose monthly income is $100. Initially, the price of a
composite good Y is $1 per unit and the unsubsidised price of a unit of good X is
$4 per unit. When a subsidy of $1 per unit is given on purchases of X, George buys 9
units of X per month. If, instead of a per unit subsidy on X, the government gives George
2 units of X free of charge, will George necessarily be worse o in this case compared to
the case of the subsidy? Explain your answer.
Solution
Start: 4x + y = 100
Unit subsidy: 3x + y = 100, x = 9 => y = 73, budget constraint : y = 100 3x.
Lump subsidy: 4(x2)+y = 100, y 100, x 2, budget constraint : y = 1084x
Unit subsidy optimal bundle is not aordable under Lump subsidy, because 4(9 2) +
73 = 101 > 100. Constraint 108 4x > 100 3x => x < 8 corresponds to Lump subsidy
bundles, lying on budget constraint and not aordable under Unit subsidy: for example
bundle (2,100). Thus we can't say if individ is better or worse.
4. Suppose that a per unit subsidy on a good which is being consumed is replaced
by an in-kind transfer of the good of equivalent money cost. Analyze the impact of this
policy on individual's welfare.
Solution
Individ will be not worse, because the best one of unit subsidy bundles will be still
aordable, while new bundles, for example, where all money (m + s x ) spended on
some good y, aordable exclusively for in-kind transfer.
5. The economy N is about to join the European Union. As a consequence the price
of milk will rise to eight times its pre-entry value but the price of wine will fall by fty
per cent. Economists of country N have estimated consumer demand in the republic and
have concluded that it is closely approximated by the demand generated by the utility

1
function u = xa1 x1a
2 with 0 < a < 1. They further estimate that the people of country
N spend more than three times as much on wine as on milk. They conclude that entry
to the European Union is in the interests of economy N. Are they right?
Solution
8p1 x1 + 21 p2 x2 = m
p2 x2 3p1 x1
Unsing Lagrange method achieve ordinary demand function:
x1 = am p1
, x2 = (1a)m
p2
v(p1 , p2 , m) = ( p1 ) ( (1a)m
am a
p2
)1a
1
If utility increased, then v(8p1 , p2 , m) > v(p1 , p2 , m) => (
2
am a
8p1
) ( 2(1a)m
p2
)1a >
( am
p1
)a ( (1a)m
p2
)1a => ( 18 )a (2)1a > 1 => 23a 21a > 1 => 214a > 1 => a < 41
1 1 2(1a)m am
Because of relative quantities and prices: p2 x2 > 38p1 x1 => p2 > 24p1 8p =>
2 2 p2 1
1 1
1 a > 3a => 1 > 4a => a < 4 . Because a (0, 4 ) that means they are right.
6. Rai spends her income on fuel for heating her house (X) and a composite of all

other goods (Y). Her preferences are represented by the utility function u= x + y.
The price of the composite good is 1, and the price of heating fuel is p. Let m denote
Rai's income.
(a) Derive Rai's demand for X and Y.
Solution
d x y p
Using Lagrange method or the fact that
d y
=
x
= 1
=> y = xp2 => xp2 + xp = m
m mp
we achieve x = , y =
p(p+1) p+1
(b) Suppose m=80 and the unit price of fuel is p=4. The energy company oers Rai
the option to switch to a dierent tari. Under the new tari, Rai must pay a xed fee
of F and then she can buy fuel at a unit price of 3. Find the maximum F that Rai is
willing to accept (and switch to the new tari ).
Solution
80
She must get at least as much, as she get without tari. It means, that ( 4(4+1) )0.5 +
( 804
4+1
)0.5 = ( 3(3+1) ) + ( (80F
80F 0.5
3+1
)3 0.5
) => F = 5 where to solve comfortable take k=80-F.

(c) The government decides to give Rai a heating fuel subsidy of s per unit. This
results in an increase in utility. Could the government follow an alternative policy that
would result in the same increase in utility for Rai, but cost the government less? Explain
using a suitable diagram.
Solution
m m(ps) 0.5 m+f 0.5
v(p s, 1, m) = v(p, 1, m + f ) => ( (ps)(ps+1) )0.5 + ( (ps)+1 ) = ( p(p+1) ) +
( (m+f
p+1
)p 0.5 m
) , f < ( (ps)(ps+1) m
s). Lets say f = ( (ps)(ps+1) s)e, then v(p, 1, m+f ) =
m m
m m+( (ps)(ps+1) s)e (m+( (ps)(ps+1) s)e)p
v(p, 1, m+( (ps)(ps+1) s)e) = ( p(p+1)
)0.5 +( p+1
)0.5 and
m m
m+( (ps)(ps+1) s)e (m+( (ps)(ps+1) s)e)p
m m(ps) 0.5
( p(p+1)
)0.5 + ( p+1
)0.5 = ( (ps)(ps+1) )0.5 + ( (ps)+1 ) . If
solution for e
exists, then some lump-subsidy policy can be emplemented.
7. Maya consumes only two goods, chocolate and clothes. Her income consumption
curve for chocolate comes out as a vertical line when plotted with quantity of chocolate

2
on the horizontal axis and quantity of clothes on the vertical axis. This implies that
chocolate might be a Gien good for Maya. Is this true or false? Explain carefully,
dening relevant economic terms.
Solution:
It is false, because the good is neutral thus not inferior and thus can't be gien.

8. Why fall in a price of a good may decrease the quantity of good consumed while price
discount oered to the same consumer for the units in excess of current consumption
results in an increase in a quantity of good consumed?
Solution:
Start: px + y = m, optimal x0
Price fall: (p s1 )x + y = m, optimal x1 < x0
Excess (p s2 )(x x0 ) + y = m px1 => (p s2 )x + y = m px1 + (p
s2 )x0 , optimal x2 > x0
So we can treat situation with excess like decrease both in income and price. Because
increase in income leads to decrease in consumption of good x it must be inferior.
Substitution eect always negative while income eect is negative for inferior good and
in order to get positive total eect this income eect must be big enough to owerweight
substitution eect. Consequently we deal with Gien good.
9. Assume that Peter buys two normal goods, X and Y. Assume further that his
income doubles and the price of X triples. In response, Peter increases consumption of
X. Thus, we can infer that Peter is worse o after the changes. True or false. Explain.
Solution:
In period 0: px x + py y = m
0
0 px ux0
M RSx,y = py
= u0y0
In period 1: 3px x + py y = 2m => 23 px x + 21 py y = m
0
1 3px ux1
M RSx,y = py
= u0y1
0 0
1 0 ux1 ux0
M RSx,y > M RSx,y => u0y1
> u0y0
by condition of increased x consumption we know that

x0 < x1 and using diminishing marginal utility assumption (weaker then diminishing MRS)
0 0 0 0
ux1 < ux0 => uy0 < uy1 => y 1 < y 0 that means Peter consumes more of appreciated
good and less of good with unchanged price while his income not changed. Consequently

3
this bundle was aordable before and not purchased thus Peter become worth.
10. A consumer is paid each week 4 units of good 1 and 4 units of good 2. These units
could be consumed or trade at market prices and both goods are innitely divisible. In
the rst week he consumed 5 units of good 1 and 3 units of good 2. In the second week
the prices were dierent and he consumed 6 units of good 1 and one unit of good 2.
(a) Find the market prices for each week.
(b) In which week is the consumer better o ?
(c) Is good 1 an inferior good for this consumer?
(d) Is good 1 a Gien good for this consumer?
Solution:
a) 4p01 + 4p02 = 5p01 + 3p02 => p01 = p02
4p11 + 4p12 = 6p11 + 1p12 => p11 = 32 p12
0 0 0 0 1 1 1 1
b) 6p1 + p2 < 4p1 + 4p2 and 5p1 + 3p2 > 4p1 + 4p2 thus 1) bundle aordable in period
0) and not choosed thus 0)>1) and preferences do not contradict WARP.
d(x1 )
c) Using Slutsky equation:
d(p1 )
= d(h1)
d(p1 )
(4 6) d(x
d(m)
1)
= d(h1)
d(p1 )
+ 2 d(x
d(m)
1) 1 0
. Because p1 > p1
in ratio terms and this increase leads to increase in x1 consumption it is evidient that
d(x1 )
d(p1 )
> 0 => d(x 1)
d(m)
> 0 => good is inferior.
d) Can't say denitely because we don't now money income.
11. Explain, why an increase in the basic wage rate per hour oered to a worker may
decrease the number of hours she wishes to work while an overtime premium oered to
the same worker may increase the number of hours she wishes to work?
Solution:
Budget under wage increase: c + l(w + k) = a + j(w + k)
Budget under overtime premium: c + l(w + o) = a + j(w + o) T o, if j > T ,
where T is time after which premium provided. l(w+o) because person already works
more then T thus he loose money with premium when lazy.
Suppose k=o, then under wage increase person has To additional money thus because
of diminishing MRS he will work less then if he has overtime premium. This can explain
the phenomenon.
12. Consider a two-period intertemporal choice model.
() Suppose there is an increase in person's wealth while the interest rate remains
constant. Will person's current saving necessarily fall if consumption in each period is
normal good? Explain.
Solution:
Savings will not neccesseraly fall because additional wealth can be destributed between
two periods.
(b) In a two-period intertemporal choice model can a net lender become a net borrower
as a result of an increase in the interest rate? Explain.
Solution:
He can't because his bundles will be not better then aordable but not choosed before.
Intuitively as a result of increase in interest rate remeaning lender he will increase his
income while becoming borrower he will pay even more then before for the same amount
of good.

4
13. Mary expects her future earnings to be worth $100. If she falls seriously ill, however,
Mary's expected future earnings are only $49. She believes that the chance of falling ill

is 1/3. Mary's utility function is u(y) = y , where y stays for earnings. Suppose that an
insurance company oers to insure Mary against loss of earnings caused by illness. What
is the dierence between the amount Mary would be willing to pay for full insurance and
the amount she would actually pay if she was able to buy full insurance on actuarially
fair terms? Provide analytical solution, graphical solution in contingent commodities
space, and explain intuitively.
Solution:
Suppose L is maximum amount of money Mary willing to pay.
1
3
u(49) + 23 u(100) = u(100 L)
1

3
49 + 23 100 = 100 L
L=19
1
Mary's expected wealth is
3
49 + 23 100 = 83.
It will not change if she will give 100-83=17 for insurance and it is will be fair term
F=17. Thus the dierence is L-F=19-17=2.
Suppose s1 is healthy and s2 is ill.

Indierence curve here is u = 13 x1 + 32 x2 => x2 = 94 (u 13 x1 )2

14. Suppose that a risk averse person is oered to purchase insurance but insurance
premium is unfavorable. If he purchased partial insurance at this premium, how would
his demand for insurance change if the probability of accidents goes up?
Solution:
Suppose a - cost of one dollar of insurance (a<1 dollars pay and 1 dollar recieve if
accident accures), L - looses in case of accident, I - money spend on insurance.
Eu = pu(w aI) + (1 p)u(w aI (L I)) = pu(w aI) + (1 p)u(w + (1 a)I L)
d(E(u))
Maximizing utility
d(I)
= apu0 (w aI) + (1 a)(1 p)u0 (w + (1 a)I L) = 0 =>
ap
u0 (waI+(IL)) ap d( (1a)(1p) ) a
u0 (waI)
= (1a)(1p)
. And because
d(p)
=
(1a)(1p)2
> 0 (because a<1),
u0 (waI+(IL))
when p increase probability of accident (1-p) decrease thus decline. Because
u0 (waI)
d(waI+(IL))
risk averse undermines diminishing marginal utility (rst derivative)
d(I)
>
u0 (waI+(IL)) d(waI) u0 (waI)
0 => d(I)
< 0 and d(I) < 0 => d(I) > 0. Thus increase in I will

5
u0 (waI+(IL))
denitely decrease . And because this decrease is a result of accident
u0 (waI)
probability increase the last must be accomponaited by increase in I.
15. Consider Mary from problem 13. Suppose that insurance is not available but Mary
can pool risks with Michael who faces an identical but independent risk that he could
fall ill and his earnings fall from $100 to $49.
(a) Will Mary agree to pool risks with Michail?
Solution:
1
Initial uxpected utility is
3
49 + 23 100 = 9
1
q
With Michael expected utility of Mary will be
3
13 49 + 2 13 32 100+49
2
+ 32
2

3
100 9.06
So expected utility of Mary has increase thus she will agree
(b) Will Michael agree to pool risks with Mary if he is a risk lover?
Solution:
For risk neutral both alternatives will be equal because of equal expected utility that
is 83. If he ir risk lover he will desagree because it will decrease his expected utility.
16. In a production process, is it possible to have decreasing marginal product for each
input and yet increasing returns to scale? If yes, then provide an example. If no, then
prove that it is impossible.
Solution:
F (L, K)L0.9 K 0.9 then F (tL, tK) = t0.9 L0.9 t0.9 L0.9 = t1.8 L0.9 K 0.9 >
Suppose function
0.9
tL K > F (L, K). Where M PL = M PK = d(F
0.9 0.9 )
d(L)
= 0.9 K10L .

17. Consider a rm with production function F (K, L) = KL + K . Then its long run
average cost does not depend on the amount of output produced. True or false? (Note:
do not calculate AC)
Solution:
F (tK, tL) = tKtL + tK = t2 KL + tK = t KL + tK = t( KL + K) = tF (K, L)
thus we have constant return to scale. Lets minimize LAC (similar as to minimize total
wL+rK
costs) for some abstract constant return to scale function. min( ) on F = q and
q
q wL+rK q
suppose minimization for output we will get min(
t q ) on F = t => min( wtL+rtK
q
) on F (tK, tL) =
t
q. Solving for tL and tK give the same result for this second mimimization problem as
to rst thus LAC independend of output q.
18. A rm's production function is given by Q = min(K 2 , 4L2 ) where Q is the quantity
of output per period of time, and L and K are inputs of labor and capital services per
period of time. Labor and capital services can be bought at constant prices per unit of
$16 and $1 respectively.
(a) Derive the long-run average cost function, illustrate the LRAC and explain the
shape.
Solution:
min(K 2 , 4L2 ) (
Maximize
2 2
on K + 16L = c. Thus K = 4L => K = 2L => 16L +
c
L = 18 c 2 c 2 c2
2L = c => 18L = c => c
thus Q = min(( ) , 4( ) ) =
9 18 81
=> LRAC =
K=9

6
c c 81 c2

Q
= c2
= c
. Now lets derive LRAC as function of Q.
81
= Q => c = 9 Q thus
81
81
LRAC(Q) = 9 Q
= 9Q . The shape is dawnward sloping because of increasing returns
to scale.
(b) Derive the short-run average cost function, illustrate several SRAC curves and
explain the shape.
Solution:
Here Q will be constant while SRAC will increase thus here SRAC curve is vertical
c
2
line. If xed factor is K then Q = K and SRAC =
Q
= K+16L
K2
.
19. True or false?
(a) In the SR a competitive rm would never produce where the AVC curve is
downward sloping.
Solution:
Yes it is true because demand curve starts after intersection of MC and AVC where
AVC become upward sloping. While AVC is dawnward every next unit of production
will yield more then previous one thus maximum is not achieved yet.
(b) In the SR a competitive rm would never produce where the SRAC curve is
downward sloping.
Solution:
d(SRAC) d(AV C)
SRAC = AV C + AF C => d(q)
= d(q)
+ d(AF C)
d(q)
where
d(AF C)
d(q)
<0 and suppose
d(SRAC)
d(q)
< 0 but d(AV C)
d(q)
inappropriate and when it is positive the statement is false

because maximization task depends only on variable costs: max(pq-VC(q)-FC).



For example AV C = q q => d(AV C)
d(q)
= 1 21 q and AF C = FqC => d(AF d(q)
C)
=
2 1.5
Fq2C then d(SRAC)
d(q)
= d(AV C) d(AF C)
d(q)
+ d(q) and if q=4 and FC=100, M C = d(V C)
d(q)
= d(q +qq
)
=
d(AV C) d(AF C)
2q+1.5 q = 11 thus p=11 (maximization point) d(q) = 1 21 4 = 0.75 > 0 and d(q) =
F4C2 = 100
16
= 6.25 from these we get d(SRAC) d(q)
= 0.75 6.25 = 5.5 < 0 thus the
statement is false.
2
20. Consider a rm with the following cost function C(Q) = Q + 3Q + 25.
a) Find the long-run supply and the short-run supply of the rm, under the assumptions
that the total cost function is the same in the long and in the short run, but the xed
cost is sunk in the short run.
Solution:
2 p3
Solving the task max(Qp Q 3Q 25) we get Qs = it is Short-Run supply.
2
p3 p3 2 p3
Prot
2
p ( 2 ) 3 2 25 > 0 if p>13 thus in Long-Run rm will remain on
the market only for this price (Long-Run supply equals Short-Run) else it will leave the
market and have zero supply. In Shor-Run rm will not leave the market because xed
costs are sunk.
b) The inverse demand function for the product of a perfectly competitive industry is
given by Qd = 250 10p, where Q is the total quantity. Find the long-run equilibrium.
Solution:
In equilibrium N Qs = Qd thus N p32
= 250 10p. Because of zero prot condition
133
p=13. Consequently N = 250 10 13 => N = 24 and Q = 250 10 13 = 120.
2
Thus equilibrium is 24 rms producing 120 units of output.

7
c) Suppose that a tax of 2 per unit sold is now imposed on every unit sold. Calculate
the deadweight loss from the tax assuming that the number of rms in this industry
stays the same (i.e. look at the SR).
Solution:
2 p5
After tax solving max(Q(p 2) Q 3Q 25) we get Qs = consequently zero
2
p5 p5 2 p5 p5
prot condition
2
(p2)( 2 ) 3 2 25 = 0 => p = 15 thus N 2 = 25010p =>
155
N 2 = 250 10 15 => N = 20, Q = 100.
R 5 R 25 Q R 5 R 25 Q
Deadweight loss = 24 10
dQdP 20 10
dQdP = 535
0 2Q+3 0 2Q+5
21. Consider constant cost industry that is in the long run equilibrium. Assume that
a per-unit tax of $2 is levied and the current tax-inclusive long-run equilibrium price
is $11. Elasticity of demand is -4. Would a small increase in the tax rate raise extra
revenue in the long run?
Solution:
dp dQ
In equilibrium p = M C + t => dt
= 1 =>
dt
= dQdp
dp
dt
= dQ
dp
. Also supply equals
d(Q) 11
d
demand and in equilibrium Q = Q = Q thus
s
d(p) Q
= 4 => d(Q)
d(p)
= 4Q
11
= d(Q)
d(t)
=>
dQ 4Q 4Q 2 4Q
4R = (t+4t)(Q+4t dt )tQ = (t+4t)(Q4t 11 )tQ = t4t 11 +4tQ4t 11 =
4t(Q t 4Q
11
) = 4t(Q 8Q
11
) = 4t( 3Q
11
) thus revenue will increase.
22. Making appropriate assumptions, show how a combination of binding (eective)
price ceiling and a binding minimum production quota may result in zero deadweight
loss.
Solution:
If production quota or price celling equals to equilibrium values. (
600 30p, p < 20
23. Consider perfectly competitive industry with market demand Qd =
0, p 0
Suppose that this good is produced by identical rms with individual cost function
T C = 0.5q 2 Assume that there are 90 rms in the industry and new rms can't enter
this industry in the current period. Government decided to subsidize the production
of good x but only 60 out of 90 rms get the subsidy (the rms that get this subsidy
are chosen randomly and the result of this choice is announced before the production
decision is made). Subsidy constitutes 50% of the price paid by consumers and it is
nanced by a lump sum tax (the same for all consumers). Suppose that entry/exit is
impossible in the current period.
(a) Find the resulting equilibrium.
Solution:
For each rm p=MC thus p = q1 , 1.5p = q2 => q = q1 30+q2 60 = 30p+90p = 120p
thus 600 30p = 120p => p = 4
(b) Calculate the resulting gain/loss in total surplus. Comment on the sources of
eciency loss/gain.
Solution:
1 (430+660)2
T Sa = 30
+ 30 (4 4 0.5 42 ) + 60 (4 6 0.5 62 ) = 4400
2
1 (590)2
Under perfect competition we have p=q=5 thus T S =
30
2 +90(550.552 ) =
4500

8
DW L = T S T Sa = 4500 4400 = 100
24. What is worse than a monopoly? A chain of monopolies A monopoly manufacturer
of an intermediate good sells a quantity q to a monopoly retailer. The retailer faces
demand q=1-p, where p is the nal price. The retail cost is , and the manufacturer's
cost is C=cq, where 0<c<1.
(a) Find the retail price under decentralized structure and compare it with the retailer
price under vertical integration.
Solution:
Under decentralized structure retailer will maximize his prot pr (1pr )(1pr )pm
1+pm
and set pr = while manufacturer prot is (1pr )pm c(1pr ). Because retailer price
2
1+pm
depends on manufacturer we get (1
2
)pm c(1 1+p
2
m
) thus pm = 1+c
2
consequently
1+c
1+
pr = 1+p
2
m
= 2 2 = 3+c 4
Under vertical integration here is no manufacturer price and rms maximize their
1+c
total prot (1 p)p c(1 p) thus p =
2
(b) Compare the prot of integrated rm with the total prot of retailer and intermediate
good seller under decentralized structure.
Solution:
Prot under decentrilazied structure is
3+c
4
(1 3+c
4
) (1 3+c
4
) 1+c
2
+ (1 3+c
4
) 1+c
2

3+c 3 2
c(1 4 ) = 16 (1 c) while under vertical integration 1+c 1+c
(1 2 ) 2 c(1 2 ) = 1+c
1
4
(1 c)2 and because 41 (1 c)2 > 16
3
(1 c)2 prots under vertical integration are higher.
25. A price discriminating monopolist sells both in its home market and in the foreign
market at prices ph ajd pf where pf > ph . The monopolist's marginal cost schedule is
rising and marginal revenue curves are declining on each market. How is the volume and
value of domestic sales aected by a tax per unit on foreign market sales? (Assume that
tax rate is small enough so that the monopolist still sells at both markets)?
(a) Provide graphical solution assuming that all curves are linear (you are expected
to provide some comments to the graphs).
Solution:
(b) Provide analytical solution for general case.
Solution:
Monopolists solve following prot maximization task p[ph ]qh + qf (p[qf ] t) c[qh + qf ]
thus
(
qh dp h
+ ph = M C dp dp
dqh
dpf => qh dp
dqh
h
+ ph = qf dqff + pf t => qh = dqh
(q f
dph f dqf
+ pf
qf dqf + pf t = M C
t ph ) =>dqh
dt
dqh
= dp h
> 01 thus sales at home market will increase.
26. Two rms in an industry produce a homogeneous good X and compete in prices.
The demand curve for X is Q = 20 P . Assume that the rms have constant but
dierent marginal cost: M C1 = 2 and M C2 = 4 and no xed costs. Assume that prices
are measured in discrete units (that is, it is impossible to cut the price by less than
(assume that is close to zero)). Draw the reaction functions and nd Nash equilibrium
in this Bertrand game with dierent marginal cost. Calculate the resulting prots.

1 because dqh <0


dph

9
Solution:

27. Consider an industry with inverse market demand P = 22Q. There are two rms:
rm A has cost function CA (qA ) = 2qA and rm B has cost function CB (qB ) = 6qB .
(a) Find the Cournot equilibrium. Illustrate graphically.
Solution:
Solving prot maximization tasks qA (22 qA qB ) 2qA for qA and qB (22
qA qB ) 6qB for qB we get response functions qA = 10 0.5qb , qB = 8 0.5qA and
substituting one into another we get Nash equilibrium output qA = 8, qB = 4
(b) Now consider a three-stage game, where rm A moves rst, then at second stage
rm B observes the output of rm A and chooses its own output, nally at stage 3 rm

10
A can change its mind about how much to produce and makes a nal output decision.
Find equilibrium. Compare the resulting prots with part (a) and explain the dierence.
Provide graphical solution using diagram from (a).
Solution:
A->B->A
Firm A in the end will always react with response function. Thus rm B will maximize
it's prot under knowlendge that A will use response function. Solving prot maximization
task qB (22 (10 0.5qb ) qB ) 6qB we get qB = 6 and using response function of A
we get qA = 10 0.5 6 => qA = 7
(c) Calculate the value of deadweight loss in (a) and (b). Compare and explain the
reasons for ineciency.
Solution: 2
qA
Maximum possible TS we get from solving
2
+ qA (22 qA ) 2qA where qA = 20
consequently TS=200.
2
T Sa = (8+4)
2
+ 8 (22 8 4) 2 4 + 4 (22 8 4) 6 8 = 136 => DW La = 64
(7+6)2
T Sb = 2 + 7 (22 7 6) 2 7 + 7 (22 7 6) 6 8 = 148.5 => DW La = 51.5
Ineciency appear due to rm with higher marginal costs.
28. Consider perfectly competitive industry with 2 rms producing good x: one rm
with cost functionc1 (q1 ) = 0.5q12 and the other rm with cost function c2 (q2 ) = 0.25q22 .
Let inverse demand function for good x be given by P (x) = 52 x. Assume that both
rms pollute water and consumers suer from water pollution: each additional unit of
q1 +q2
output (produced by any rm) results in losses in consumers' surplus equal to .
9
(a) Find equilibrium and represent it graphically.
Solution:
Here we deal with competitive industry where rms are price takers consequently
necceserally conditions is p=MC. So p = M C1 = q1 => q1 = p, p = M C2 = 0.5q2 =>

11
q
q2 = 2p. We also know that q = q1 + q2 = 2p + p = 3 => q = 3p => p = 3
q
Market price formation equality is 52 q = => q = 39
3
(b) Is the equilibrium allocation found in (a) ecient? Calculate the value of eciency
losses (if any) and show these losses on your graph.
Solution:
(q1 +q2 )2
Solving TS maximization task
2
(q1 +q
9
2)
+(q1 +q2 )(52q1 q2 )0.5q12 0.25q22
936
we get q1 =
31
, T S 1010, q1 13, q2 26, Q 39
So the result in (a) is very close to ecient allocation and DWL is approximately zero
but allocation found in (a) is not ecient in strickt sense.
q1 +q2
Because M EC =
9
= 9q , thus SM C = M C + M EC = q
3
+ q
9
= 4q
9
consequently
52 q = 4q
9
=> q = 36

(c) Assume that government decides to introduce direct regulation but it has no
information about the rm's type (it only knows that there is one rm of each type). If
government sets the same maximum quota [rm can produce the amount that does not
exceed this quota] equal to the half of ecient output for each rm, will the resulting
allocation be Pareto ecient?
Solution:
Half of ecient output undermines q1 = 7.5, q2 = 13.
(d) Now, suppose that government has the same asymmetric information problem as
in (c) but uses uniform tax policy instead of uniform quota. Is it possible to attain the
ecient allocation?
Solution:
29. A society consists of 2 identical individuals who derive utility from a public good.
The public good can be provided at a constant marginal cost of 6. Let xi denote the
level of public good provision by i and let X denote the total provision of the public

12
good. The net benet enjoyed by individual i from providing xi units of the public good
2
is given by Ui (xi , X) = 0.5(10 X) 6xi , i = 1, 2
(a) Derive the socially optimal level of provision of the public good.
Solution:
Social optimal provision maximize total utility. Solving maximization task (10x1 +
x2 )2 6x1 6x2 we get U = 51 and x1 + x2 = 7
(b) If every individual optimally chooses how much public good to provide [individuals
make their decisions independently and simultaneously], derive the total level of provision
of the public good.
Solution:
First lets nd reaction functions of individuals. Solving maximization task 0.5(10
x1 +x2 )2 6x1 for x1 we get x1 = 4x2 and because individuals are identical x2 = 4x1
consequently Nash equilibrium requires x1 + x2 = 4 thus X=4
Solution:
(c) Suppose n>1 new individuals arrive in the society. The net benet enjoyed by
new individual j from providing xj units of the public good is given by Vj (xj , X) =
13 (9 X)2 6xj . Suppose every individual optimally chooses how much public good
to provide. Does the total level of public good provision change compared to part (b) as
a result of the new arrivals? Explain your answer.
Solution:
I. If new individuals substitude previous two
1
Pn 2
P
Maximizing (9 x1 j=2 xj ) 6x1 we get response function x1 = j=2 xj
3
consequently the only possible equilibrium is x1 = ... = xn = 0 thus X=0.
II. If new individuals are add to previous two
From I) we know that new invididuals always response with zero production consequently
equilibrium will be the same as in (b).
30. There are two kinds of workers in community: high-skilled with marginal revenue
product of $1200 per month and low-skilled with marginal revenue product of $600
per month. Number of low-skilled is two times bigger than number of high-skilled. It
is too expensive to monitor anybodies' work. The community college oers a course
in microeconomics that doesn't change the workers' productivity. High-skilled workers
think that an hour lecture is as bad as loosing $20 and for low-skilled an hour lecture is
as bad as loosing $50. All rms in this community are risk neutral and pay wages equal
to the productivity of an average citizen of this community. The new management of
rm XYZ decides to change the compensation scheme. Which of the following strategies
would be most protable for XYZ (assuming that other rms do not change the wages):
a) wage of $1000 and require workers to listen to 3 hours of lectures per month,
b) wage of $950 and require workers to listen to 2 hours of lectures per month,
c) wage of $900 and require workers to listen to 3 hours of lectures per month,
d) wage of $850 and require workers to listen to 2 hours of lectures per month,
Explain the idea of this pricing scheme.
Solution:
1
Strategie (d) will be the most protable because in other rms all workers get
3

2
1200 + 3 600 = 800$ while in XYZ bad workers will recieve 850-2*50=750$ so less

13
then in other rms while good workers will have 850-2*20=810$. Thus good workers will
choose XYZ over other rms (because XYZ gives higher wage) while bad workers will
choose not to deal with this rm at all.
31. Consider the market for second-hand gadgets. There are 200 sellers with good
quality gadgets and 400 sellers with bad quality ones. Assume also that there are 700 of
potential buyers (each buyer is willing to purchase one gadget). A good quality gadget
is worth $104 to the seller, while a bad quality one is worth only $80 to the seller. A
buyer is willing to pay $130 for a good gadget, and $100 for a bad one. Each seller knows
the quality of his gadget but buyers are not able to distinguish a good one from a bad
one. All agents are risk-neutral. Assume that all this information is common knowledge.
Each seller can sell the gadget together with a guarantee that promises to repair the
gadget in case of electronic failure. This guarantee costs $X to the quality sellers, and
costs $Y to the bad ones. Find all possible values of X and Y under which signaling is
successful.
Solution:
If X>20 then bad sellers will not provide guarantee at all because it will give them
100-80-20 negative prot. Consequently if X 20, Y 20 signaling must be eective.
32. Suppose an agent borrowers funds and invests in a project. His eort must be
monitored by lenders to ensure that investment is successful, and monitoring is costly.
If the agent borrowers from several lenders, he is likely to be monitored at an inecient
level. Is this true or false? Explain your answer using the appropriate model.
Solution:

2 Quizes
2.1 First quiz

1. Eric spends all his monye on herring potatoes. When the price of potatoes was 9
crowns per unit and the price of sh was 5 crowns per unit, he spent his entire income
to buy 5 units of potatoes and 10 units of sh per month. Now the government starts
subsidizing potatoes: market price haven't changed, but consumers get a subsidy of
5 crowns per each unit of potatoes consumed. To pay this subsidy, the government
introduce an income tax. Eric pays an income tax of 20$ per month. If P denotes the
number of units of potatoes and F denotes the number of units of sh, what is Eric's
new budegt line equation?
Solution:
His income is m = 95+510 = 95. After subsidy we have (95)P +5F = m20 =>
4P + 5F = 75
2. When the prices were (5,1) the consumer chose the bundle (x0 , y0 ) = (6, 3). At
the new prices (px , py ) she chooses the bundle (x1 , y1 ) = (5, 7). For her behavior to be
consistent with the weak axiom of revealed preferences, it must be that:
Solution:
Income is 5*6+1*3=33. New bundle under old prices costs 5*5+7*1=32 it is less then

14
income (32<33) and thus it is worse then old (because new bundle was aordable under
old prices while old bundle has been choosed). Consequently an old bundle must not be
aordable under new prices (else it will be choosed) thus x0 px + y0 py > x1 px + y1 py =>
6px + 3py > 5px + 7py => px > 4py
1
3. There are tuility fynctions: uA = xy, uB = 1000xy, UC = xy, uD = , uE =
xy+1
xy 10000uF = x/y, uG = x(y + 1) which of them are the same?
Solution:
Calculating M RSx,y we get M RSA = xy , M RSB = 1000x
1000y
= xy , M RSC = xy , M RSD =
(xy+1)2 y 2

(xy+1)2 x
= xy , M RSE = xy , M RSF = xyy , M RSG = y+1 x
we see that A,B,E,D are
similar.
4. Pamela consumes only two goods - x and y. Her income doubles and the prices of
the two goods remain unchanged. Assuming that she is a utility maximizer and likes
both goods what will happend?
Solution:
She will consume more of both goods because of diminishing MRS.
5. Joes utility function is u=xy. He consumes 4 units of x and 16 units of y. How much
units will he be willing to give of y for a small amount of x.
Solution:
y
M RSx,y = x
=> M RSx,y (4, 16) = 4 thus for a small amount of x Joe will be willing
to give 4 units of y.

2.2 Second quiz



1. Alena's utility function is U (x, y) = 2 x + y . Under her current income and prices,
she consumes 5 units of x and 20 units of y. If her income doubles, while prices stay
constant, how many units of x will she consume after the change in income?
Solution:
Because utility function is quasilinear she will always consume the same amount of
p1
good x. In order to proof lets look at utility maximization condition M R =
p2
=> 1x =
p1
. Because increase in income did not change price relation (or we can think about this
p2
as proportional decrease in all prices) the amount of x consumed will remain the same.
2. Ahmed consumes only caviar and bread; bread is inferior good for him. The price of
caviar falls, but there is a reduction in his income that keeps him on the same indierence
curve as before (Peter's indierence curves as smooth).
Solution:
The fact that bread is inferior good do not matter in this task. Due to decrease in
caviar price M RS will decline and because indierence curve is the same MRS will be
less in point where more caviar and less bread.
3. In 2013 Pavel spent his income on x and y. Between 2013 and 2014, the price of x
rose by 8% and the price of good y rose by 8%. In 2014 Pavel bought the same amount
of x as in 2013, but he bought more of good y thant in 2013. From these facts, could we
conclude that x or y is inferior good?
Solution:

15
This situation is similar to decline in income while prices are unchanged. Decrease in
income result in increase of y consumption thus it is inferior good.
4. Bellas thinks red roses and white roses are perfect substitutes, one-for-one. If red
roses currently cost 2$ per unit and white ruses cost 3$ per unit, and if the price of red
roses rises to 4% per unit whaat will happend?
Solution:
The entire change in demand for red roses will be due substitution eect.
5. Elizabeth consumes only yoghurt and broccoli. When the price of yoghurt rises with
no change in her income or the price of broccoli, Elizabeth buys less broccoli and less
yoghurt. From this information we can denitely conclude that.
Solution:
Because consumption of broccoli decline we can conclude that negative income eect
was stronger than positive substitution eect. Consequently brocolli can't be inferior so
it is normal good.

2.3 Third quiz

1. Penelope has waited a long time for her ship to return, and she has concluded that the
1
probability that it will return today is . If the ship does come today, she will recieve 16$.
4
If it doesnt't come today, it never will and she will have zero wealth. She is an expected
utility maximizaer with Neuman-Morgenstern utility function equal to the square root
of her total income. What is the minimum price at which she would sell the rights to
her ship.
Solution:

1
u= x => Eu =
4
16 + 43 0 = 1 and expected utility equals to the minimum price
at which she is willing to sell her rights.
2. Alex likes to gamble. Donna oers to bet him 40$ on the outcome of a boat race.
If Alex boat wins, Donna would give him 40$. If Alex's boat doesn't win, Alex would
give Donna 40$. Alex is an expected utility maximizer with a von Neuman-Morgenstern
2
utility function u(x) = x . If Alex's total wealth before the race is 60$ and he belives
that the probability of his boat to win is 0.4 what will be the eect of the bet on his
expected utility?
Solution:
His initial utility is uinitial = 602 = 3600. If he bet Eubet = 0.41002 +0.6202 = 4240.
Because Eubet > uinitial taking the bet would increase his expected utility.
3. Person prefers C to B. What to what he also denitely prefers?
4. If the person is risk-loving how will change the answer to the previous question?

2.4 Fourth quiz

1. A competitive rm produce gift-wrapped pieces of Berlin wall. The production function

is y= K + L, where y is the number of pirces produced, K is capital and L is labour.
Neglect the use of the wall itself. The price of capital is r and of labour is w. How prices
would aect K and L values?

16
Solution:
Because of production function structure if r>w it is protable to use only L, if w>r
then only K and if w=r it is no dierence how much of which production factor to use.
p
2. Yuri's production function is F = min(x1 4x2 ). If the price of factor 1 is w1 = 3
and price of factor 2 is w2 = 12, then what is his supply function?
Solution:
Because production maximization under min function requires x1 = 4x2 we get q =
2 2
x1 = 4x2 => x1 = q 2 , x2 = q4 thus we have costs function 3q 2 + 12 q4 = 6q 2
2 p
consequently solving prot maximization task pq 6q we get p = 12q => q =
12
2
3. A rm has long run costs function C(Q) = 7Q + 112 if Q>0 and C(0)=0. In the
long run, it will supply a positive amount of output, so long as the price is greater than
which value?
Solution:
No reason to produce will be under p=AC because thus prot will be zero. Consequently
2
we have p = 7Q
Q
+ 112
Q
= 7Q + 112
Q
2
. Solving prot maximization task pQ 7Q 112 we
112
get supply function p = 14Q. Combining both conditions we get 7Q + = 14Q =>
Q
Q = 4 => p = 56.

4. A rm has the production function Q = x 1 x2 . In the short run it must use
exactly 10 units of factor 2. The price of factor 1 is 40$ per unit and the price of factor
2 is 6$ per unit. Find rm's short-run marginal costs function.

In short run because of xed second fuctor we have production function Q= x1

17
Q 2 Q 2
10 => x1 = 100 thus costs function is 40x1 + 6 10 = 40 100 + 60 = 0.4Q2 consequently
2
d(0.4Q )
M C = d(Q) = 0.8Q
5. In a certain country, demand function for bread was D=381-3p, and supply function
was S=5+7p, where p is price per loaf of bread. The government prohibited bakers from
selling bread for a price above 32 per loaf. To avoid shortages, the government agreed to
pay bakers enough a subsidy for each loaf of bread so as to make supply equal demand.
How much would the subsidy per loaf have to be?
Solution:
381-3*32=5+7*(32+s) => s=8
6. Consider a person Bernoulli utility function u=2x, who thinks of two assets: Greek
bonds and UK bonds. If he invest in UK, his wealth is 800. If he invest in Greece, and
Greece doesn't default, his welath will be 1000. If Greece defaults, his welath will be
zero. If the probability of Greece defaulting is 0.2, what is the maximum amount this
person will pay to nd out for certain, whether Greece will default or not?
Solution:
His initial wealth isw = 2 800 = 1600. If he will have information Eu = 2 1000
0.8 + 2 800 0.2 = 1920. In order to be with information as well as in initial state he
must loss 1920-1600=320 of his utility. Consequently 320=2*x => x=160.

2.5 Fifth quiz

1. The demand curve for the output of a certain industry is linear, q=A-Bp. There are
constant marginal costs of C. For all values of A,B and C such that A>0; B>0; and
0<C<A or B what will be the relation between competitive and monopoly output?
Solution:
Aq
We know that p=. In conmpetitive case p = M C = C => qcompetitive = A BC
B
(Aq)q
d( B )
while in case of monopoly M R =
d(q)
= A2q
B
, M R = M C = C => A2q
B
= C =>
ABC qcompetitive
qmonopoly = 2 . Finnaly qmonopoly = 2 consequently competitive output twice greater
then monopolistic.
2. A prot-maximizing monopolist faces the demand curve, q=300-3p. It produces at
a constant marginal cost of 20$ per unit. A quantity tax of 10$ per unit is imposed on
the monopolist's product. Find price of the monopolist's product.
Solution: d(q 100q )
Before tax we have p = 100q
3
=> M R = 3
d(q)
= 100
3
2q
3
, M R = M C =>
100 2q
3
3 = 20 => 20 => p1 = 80 3
. After tax M Cnew = M Cold + tax = 20 + 10 = 30
100 2q
consequently
3
3 = 30 => q = 15 => p2 = 95 3
. Finally 4p = p2 p1 = 5
3.Wobble's Weebles is the only producer of weebles. It makes weebles at constant
marginal costs c (where c>0) and sells them at price of p1 per weeble in market 1 and
at price of p2 per weeble in Market 2. The demand curve for weebles in Market 1 has
a constant price elasticity equal to -1.5. The demand curve for weebles in Market 2 has
a constant price elasticity equal to -3. Find the ratio of the prot maximizing price in
Market 1 to the prot maximizing price in Market 2.

18
Solution:
p1 1 1 1 23
p2
= 1
1 1
= 1 31
=2
2
4. In market with inverse demand curve P=6-Q, brand is monopolist with no xed
costs and with marginal costs of 2. If monopolist can use two-part tari, what is the
highest prot of the monopolist?
Solution: R4
p = M C = 2 => 2 = 6 Q => Q = 4 => = T S = 0
(6 Q)dQ = 8
5. In market with inverse demand curve P=6-Q, brand is monopolist with no xed
costs and with marginal costs of 2. Find the value of deadweight loss due to monopoly
power.
From previous task we know that maximum possible TS=8. Under monopolu we have
R2
M R = M C => 6 2Q = 2 => Q = 2 => T S = 0
(6 Q) = 10 => 4T S = 8 6 = 2

3 Home assigments
3.1 First home assigment

Short questions (a) (1 p.) The price of food rises by 10% and consumer's income by
5%. A person initially spending half their income on food would be neither worse o nor
better o as a result of these changes. Analyze carefully this statement.
Solution:
Intuitively because old consumption bundle is aordable (as well as some new) consumer
can't become worth and even might become better.
Lets denote food x, its price p and aggregate good y
Budget constraint before changes (initial): px+y=m. Index this period as 0.
Budget constraint after changes: 1.1px+y=1.05m. Index this period as 1.
m m
Initial consumption bundle was ( , ).
2p 2
Algebraically clear that under new constraint old optimal amount of consumption x
will allow to consume exactly an old amount of : 1.1p
m
2p
+ y
y = 1.05m => y = 0.5m
thus an old optimal bundle aordable under constraint and it lies in new budget line. It
proofs that consumer can't become worse. Additionaly because there are some bundles
not aordable before consumer may become even better.
Example when person become better
Under any values of m>0, p>0 and Coub-Douglass utility function person will become

better. Because consumption of x is ( ) then =


q
m 1 mm
2p 2
and thus u0 =
2p 2
= 2m and
p
q
1.05m 1.05m
u1 = 2.2p 2
= 1.001136 2m

p
consequently u1 > u0 and thus person become better.

Example when person neither worse o not better o


y
Consider some variation of Leontiev utility function u(x, y) = min(x, ) then u0 =
p
m
u1 = 2p . Note, that p
here denotes scalar that equal to the initial price of , not the x
price parametr.
To nish the task lets provide graphical interpretation below.

19
(b) (1 p.) In order to aid the poor, the government introduces a scheme where the
rsta kg of butter a family buys is subsidized and the remaining amounts are taxed.
Consider a family which consumes butter and is made neither better o nor worse o
as a result of this scheme. The total amount of tax it pays cannot exceed the subsidy it
receives. Is this true or false? Explain.
Solution:
Intuitively this statement is true because if amount of paid taxes exceeds an amount
of subsidy then the same amount of butter could be bought by lower money costs thus
lefting family with some additional money to spend on other goods.
s>0 and tax t>0, p1 = p, x1 = x, y(x2 ...xn , p2 ...pn ) =
Pn
Consider subsidy i=2 p i xi , p >
s, p > t.
(m
Initial budget: = px + y
(p s)x + y = m if xa
New budget:
m s = (p + t)(x 1) + y if x > a.
Suppouse x
is an amount of x in some bundle (not neccessary optimal) under initial
budget constraint. To proof the statment lets show that under new budget constraint
2
any bundle satisfying t(x a) > as was aordable under the old budget. Because
px (p s)a (p + t)(x a) = sa + ta tx = sa t(x a) < 0 consumer will

have less money to spend on other goods y


under new budget then under old when

amount of x is the same. Because other prices and income remain constant this reslut

exactly means that part of old budget satisyng t(x a) > as is a subset of the new
budget. Thus all this bundles was aordable before but not choosed so under assumption
of strong monotonicity they are worth and consumer can't become nor better nor even
remain the same. This proof the statement.
c. (1,25 p.) Ben can purchase all the housing he desires at price of p1 per square

2 When total amount of tax consumer pays exceed the subsidy he receives. From this evidient that x>a

20
meter. He is also eligible for housing in government project: in this case he needs to pay
p2 per square meter but must consume x2 square meters. Which option does he choose?
Explain carefully.
Solution:
Ben decision depends on preferences structure. Intuitivelly Ben maybe don't need x2
square meters and will prefer some bundle with less of them but with more of other
goods. Under another preferences structure maybe for example x2 is very close to an
exactly amount Ben need and remaining money will be enoughf to satisfy need in other
goods.
Suppose we have some bundle X that is optimal while it is no government proposition.
If bundle with government proposition will lie above indierence where X was, then Ben
will agree with proposition, else he will reject it.

(d) (1.25 p.) Consider an individual that derives utility from consumption of a
composite commodity c (with price normalized to 1) and from leisure l. This agent
currently gets a$ in form of dividends weekly and works L hours each week at w$ per
hour. Suppose we observe that if dividend income increases his consumption of aggregate
commodity always increases by more than the rise in a. If w increases this person would
increase his labour hours. True or false?
Solution:
a$ additional income he increase his labour (it is no
Intuitively if when person gets
another way to increase consumption by factor more then a) then he also may increase
his labour if his income will increase due wage rise.
Suppose L is a maximum possible ammount of work.
Budget line: c=a-lw+Lw

21
d(c)
d(a)
> 1.
d(c) d(l) d(l)
d(a)
= 1 w d(a) > 1 => d(a) < 0.
Using Slutsky equation:
d(l)
d(w)
= d(h l)
d(w)
d(l)
+ L d(a) . Because
d(hl )
d(w)
d(l)
< 0 and L d(a) d(l)
< 0 consequently d(w) < 0 thus
meaning that an increase in wage will decrease laiser that is equivalentl to an increase
in labour due to an increase in wage. So the statement is true.

War During a war, food (good 1) and clothing (good 2) are rationed. In addition to a
money price, pi (i=1,2), a certain number of ration coupons qi must be paid to obtain
good i. Each consumer has an allocation of ration coupons Q which may be used to
purchase either good, and also has a xed income m.
(a) (0,5 p.) Illustrate the budget set. Explain carefully.
Solution:
Budget constraint: p1 x1 + p2 x1 m, q1 x1 + q2 x2 Q

(b) (1 p.) Suppose the money income of a consumer is raised and he buys more food
and less clothing. It follows that clothing is an inferior good. True or false?
Solution:
This statement can be true for example if coupon constraint is a subset of money
constraint because substitution eet is zero and whole decrease is due income eect.
However possible to illustrate situation when clothing is not inferior (for simplicity also
p1
imagine that indierence curves are linear lines with slope .)
p2

22
So in general this statement is false.

(c) (1 p.) Suppose the consumer's preferences can be described by x1 + x2 .
Assume p1 = 3, p2 = 1, m = 120, Q = 90, q1 = 1, q2 = 2. Find the optimal consumption
bundle. Illustrate graphically.
Solution:
Lets solve constraint optimization x1 + x2 on 3x1 + x2 120, x1 + 2x2 90
3x+y=120 and x+2y=90 consequently x=30, y=30. Why is it maximum evidient from
graph below. Thus optimal consumption bundle is (30,30).

(d) (1,5 p.) Assume that coupons can be traded and their price is 1 per coupon.
Redraw the budget line, compute the optimum consumption bundle. Will the consumer
buy or sell coupons? How many?

23
Solution:
It is comfortable to think about this situation like at rst consumer sell all his coupons
and then buy coupons for each unit (in continious sense) of good he consume. Thus
budget constraint: (p1 + q1 )x1 + (p2 + q2 )x2 Q + m.


To nd new optimal bundle lets solve optimization problem

x1 + x2 on 4x1 +3x2
x 3 9
210. Because of MRS we get y = 4 => x = 16 y and insering this in budget constraint

we achieve an optimal bundle that is (


45
2
, 40)3 .
(e) (0,5 p.) At what price (of coupons) will the consumer not trade in coupons?
What happens at higher and lower prices?
Solution:
The question is not clear thus there are two possible approaches to answer are possible.
I. If coupons prices are qi
p1
If
m
= qQ1 and pm2 = qQ2 he will not trade. Because if so then due to linearity two
constraints are similar (thus an optimal bundle can be achieved without any trade).
II. If coupons prices are money recieved from selling coupons
If coupons trade with price a we get the next constraint: (p1 + aq1 )x1 + (p2 + aq2 )x2
Q + m. Coupons will not be traded if this (combined) budget constraint will be the
subset of intersaction of initial money and coupon budget constraints. To be so combined
constraint must be simultaneously the subset of money and the subset of coupon constraint.
( m+Q(p +aq )x (
mp1 x1 Qp2
1
(p2 +aq2 )
1 1
p2 q2 mq2 p1 x1 +q1 x1
a
m+Q(p1 +aq1 )x1 Qq1 x1 => mq2 x1 > 0
(p2 +aq2 )
q2 p2 Qp2 q1 x1 +p1 x1
a
Qp2 mq2
Thus if a max( , ) will be no trade. Else trade may be
q2 mq2 p1 x1 +q1 x1 p2 Qp2 q1 x1 +p1 x1
(depends on preferences structure).

3 Anotherway to solve the problem using strickt Lagrange method ommited (but has been
implemented) to make sollution desription shorter

24
Interest rate (2 p.) In a two-period intertemporal choice model we deal with aggregate
consumption and it is natural to treat consumption in every period as a normal good.
Given this assumption solve the following problem. Suppose a person responds to increases
in the interest rate by rst increasing saving from SA to SB and then (with further
increases) reducing saving to 0 < SC < SB . Is he necessarily irrational?
Solution:
It is absolutely not clear what further increases means. However lets interprete it
like an increase in interest rate in period two. The second unclear moment in the task
is why person make any savings in period two if it is two-period model. Thus he lefts
some part of his income unspent.
In order to avoid misunderstandings lets think about this problem like we observe
the same person in a two-period model two times: two dierent expiriments using two-
period model in each. During rst time he responds to an increase in interest rate by
an increase in savings and in second time he responds with may be decrease may be
increase in savings but if increase then it is less then in the rst time.
Savings are S = m c1 . Thus an increase in savings means decrease in c1 . So person
rst decrease c1 and then increase c1 . Graph below demostrates that person may be
rational because each time he choose some bundle unaordable before.

So the unswer is that person may be rational.

4 Second homework
First task (a) (1.5 p.) Design your own example that demonstrates the value of
additional information for a risk-averse individual. You should produce your own story
(please do not borrow the story from textbooks, exercises and other sources but think
of your own example). In addition to the verbal description, you are asked to nd the
exact value of additional information for some particular numerical example, i.e. take

25
some particular elementary utility function, probabilities, specify the outcomes and the
corresponding levels of wealth and calculate the value of additional information.
Solution
Example: Imagine some private military company (PMC)4 that recieve contracts for
airstrikes against dierent terrorist groups all over the world. These contancs complete
succesfully if and only if terrorists has no air defense: rockets, special radars and so
on. PMC calculated that the chance of pressence of air deence in average is 20%. If
contract executed succesfully PMC recieve 100$ prot (for example from government
or transnational organization client) and else get only 25$ (contract payments (they
recieve it independend of success/fail result) minus costs of downed aircraft, posthumous
payment to pilot and so on). In order to avoid air defence risk PMC can pay to spy (some
private person not PMC worker hired from the market) who will check all the contracts
and nd among them those which are not related to any air defense. Also PMC can
equip their combat aircrafts with some gadgets allowing to bypass (avoid) air deense
and thus ensure no loose but it will costs 50$.
Suppose PMC utility function is u = ln(1 + x) where x is annual prot (thus from one
contract because only one contract per year can be performed). In case of incomplete
information (uncertainty under decision act without equipment ) Eu = 0.8ln(1 +
100) + 0.2ln(1 + 25) 4.34. With additional information Eu = 0.8ln(1 + 100) +
0.2ln(1+50) 4.48. The value of additional information is 100*0.8+25*0.2-100*0.8-
25*0.2=5.
Lets nd the maximum sum PMC willing to pay x for additional information to spy.
Solving 0.8ln(1+100)+0.2ln(1+25)=0.8ln(1+100-x)+0.2ln(1+50-x) we get x 10.56.
Thus if spy services costs less 10.56$, PMC will be willing to pay for additional information.

Air defence No Air deence Expected prot Expected utility


Without equipment 25 100 85 4.34
With equipment 50 50 50 3.93
With information 50 100 90 4.48

(b) (1.5 p.) Denote by X the maximum sum that a risk-neutral person is willing to
pay for additional information and by Y - the maximum sum that a risk-averse person
is willing to pay for the same information. Suppose that X>0 and Y>0. Is it true that
X can never exceed Y?
Solution
It is absolutely unclear what the same information denitely means. Lets consider
that it means that changes in arguments of their expected utility functions (except
payments for information denoted by X,Y,Z) will be the same. So both persons dier
only in their utility functions.
Suppose ura (w), urn (w) are utility functions for risk-averse and risk-neutral individuals.
E[urn (wX)|I]Eurn (w) = 0, E[ura (wX)|I]Eura (w) = 0 where I denotes presence
of additional information. Lets rewrite it in a broad terms for an abstract person (with

4 https://en.wikipedia.org/wiki/Private_military_company

26
any utility function): E[u(w Z)|I] PEu(w) = p1 u(w1|I Z) + ... + pn u(wn|I Z)
n
p1 u(w1 ) ... pn u(wn ) = 0, where i=1 pi = 1, wj|I wj j and z, wz|I > wz . Here
wj|I wealth with additional information and wj without.
Incorrect proof that Y can never exceed X. Yes this proof is incorrect and
could be counterexemplied by the example above concerning PMC. But I
dont't know where the mistake in this proof. Please skip this part and follow
to Counterexample below if not interested 5 .
ra rn
In this case wn|I = wn|I = wn|I and wnra = wnrn = wn . Lets proof that p1 ura (w1|I X)+
...+pn ura (wn|I X)p1 ura (w1 )...pn ura (wn ) 0 that means that maximum additional
information costs for risk-neutral person will be more than maximum for risk-averse that
undermine X PYn . Because utility function of risk-neutral person is linear Pit is easy to
n
nd that X = wi ). Thus neccesserally to proof p1 ura (w1|I i=1 pi (wi|I
i=1 pi (wi|IP
wi )) + ... + pn ura (wn|I ni=1 pi (wi|I wi )) p1 ura (w1 ) ... pn ura (wn ) 0. We
know that ura is concave thus ura (
x+y
2
) ura (x)+u
2
ra (y)
=> 2ura ( x+y 2
) ura (x) + ura (y).
And because if a + e > 0, (where e 0 ) than a 0 we can rewrite p1 ura (w1|I
Pn
+ pn ura (wn|I ni=1 pi (wi|I wi )) p1 ura (w1 ) ... pn ura (wn )
P
i=1 pi (wi|I wi )) + ...
P
Pn wk|I wk i=1 npi (wi|I wi )
k=1 2pk ura ( 2
).
Lets use Jensen inequality for concave functions: f ( P
Pn Pn
i ai xi ai f (xi )
n
a
) iP
n and (note
i i i ai
wk|I wk n
P
Pn Pn p
i=1 i (wi|I w i ) P n
that
Pin pi = 1) thus we get k=1 2pk ura ( 2
) ura ( k=1 pk [wk|I
wk i=1 pi (wi|I wi )]) = ura (0) = 0. It prooves that X is always not less that Y. One
more time repeat that this proof is incorrect but I don't know why. Will be
very greatful for ding mistake.
Countrexample to the statement
Supposew = 1000, w2|I = 100, w1 = 900, w2 = 100, p1 = 0.99, p2 = 0.01, urnp=
1|I
p= w. Solving 0.99(1000X)+0.01(100X) = 0.99(900)+0.01(100) and 0.99 (1000 Y )+
w, ura
0.01 (100 Y ) = 0.99(900)+0.01(100) we get that X=99 and Y 95.25 => X>Y thus
the statement is false.
Second task (a) (0.75 p.) Suppose that individual has quadratic Bernoulli utility
2
function u(x) = x bx , where b > 0. Show that for this individual the expected utility
from a distribution is determined by the mean and variance of the distribution and, in
fact, by these moments alone. Draw the resulting indierence curves in the same axes as
in gure 5.5. [Note: to guarantee non-satiation we assume that distribution cannot take
1
values larger than .]
2b
Solution Pn Pn
2 2 2
Eu = p1 (x1 bx 1 ) + ... + p n (x n bx n ) = i=1 p i x i b i=1 pi xi
Mean is x = i=1 pi xi (mean is confusing term because not clear
Pn
if probabilities
matter thus expectation is better)
Variance 2
Pn 2 2
Pn
is = i=1 pi (xi x ) = i=1 pi x2i x2

5I start to make this proof after 8 attempts successively show me that Y>X and no one inverse. Then
I have checked it for my PMC example and only than nd that X>Y is also possible

27
Pn Pn
First we can see that b 2 = bx2 b 2 2 2
i=1 pi xi => b bx = b
2
i=1 pi xi . Now
it is easy to rewrite expected utility function:
Eu(x, 2 ) = x b 2 bx2 In order to build the graph lets nd relationship between
standard deviation and expected income. It is clear that expected income equals to
distribution mean.
1 14b(b 2 +Eu)
After simple mathematical challange (quadratic equation solution) we get x = .
2b
1 14b(b 2 +Eu) 1
But plus sign can't be used because thus x = + > . That is why nally
2b 2b 2b
1 14b(b 2 +Eu) 1
we get x = 2b
where max(x) = 2b
because else will not be real solutions.

(b) (1.75 p.) Suppose that individual considered in (a) can invest in treasury bills or
stocks or in some combination of the two. Treasury bills bring zero expected return and
are assumed to be risk-free. Stocks bring positive expected return Rs and are assumed
to be risky with standard deviation of s > 0. Suppose this investor has initial wealth
equal to 1.
(i) Derive the budget constraint in terms of mean and standard deviation of the
portfolio and illustrate it graphically.
Solution
Suppose q is a fraction person put in stock market and 1-q in treasury bills (denoted
by f symbol). Rp = qRs + (1 q)Rf let be the weighted average of the expected return
(R R )
on both assets. Thus we also get p = qs . Than Rp = Rf + ss f p and because

Rf = 0 we get budget line Rp = Rss p => budget constraint Rp Rss p

(ii) Solve the resulting utility maximization problem to get demand for risky and
risk-free assets (consider interior solution only). Illustrate graphically.

28
Solution
It is equivalent to solve optimization problem: max(Rp bp2 bRp2 ) on Rp = Rss p .
Lets use Langrange method to handle with this task but rst denoting k = Rss .
L = Rp bp2 bRp2 (Rp kp )

1 2bRp = 0 = 1 2bRp
(
2bp
= 1 2bRp n
2bp + k = 0 => = 2b k
p
=> k
=> 2b
k p
= 1 2bkp =>
Rp = kp
Rp + kp = 0 Rp = kp


( (R s)
s
n k
p = 2b(1+k2 )

p = 2b(1+( R s )2 )
( 2b
k
+ 2bk)p = 1 => k2
=> (
s
Rs 2
)
Rp = 2b(1+k2 ) Rp =
s
2b(1+( Rs )2 ) s
Using Hessian lets check whether it is maximum

2b 0 1
det 0 2b Rss = 2b + 2b( Rss )2 > 0 => maximum
1 Rss 0
Now lets nd shares (demand) q and (1-q) for these assets.
p ( R2s )
s Rs Rs
p = qs => q = s
=> q = 2b(1+( R s )2 ) = 2bs2 (1+( R s )2 ) = 2b(s2 +Rs2 )
=> 1 q =
s s
1 Rs
2b(s2 +Rs2 )
demand for risk and risk-free assets correspondingly in
and it is
share terms . Because wealth equals to 1 this shares represent demand.

(c) (1.5 p.) Suppose that individual from (b) invests in both assets. How is the
optimal portfolio aected by:
(i) an increase in coecient b?
Solution
Because b lies in the denominator its increase will result in decrease of (share q
d(q)
of risky asset) and thus increase share (demand) of risk-free. More formally
d(b)
=

s
Rs
2b2 2 (1+( (1q)
Rs 2 < 0 while d(b) =
) )
Rs
2b2 s2 (1+( R s )2 ) > 0. Intuitively it is because negatively b

s s
aects expected utility.
(ii) an increase in variance of risky asset?
Solution
d(q)
Because
d(s2 )
= 2b(RR
2
s
2 2 < 0 increase in variance of risky asset will decrease
s +s )
demand (share) for this asset and will force person to buy more of risk-free. Intuitively
it is because increase in variance undermines increase in risks for this risk-averse person

29
thus he will less satised by corresponding level of expected returns rather than with
low variance.

Third task A farmer can grow wheat or potatoes or both. If the summer is shiny then
a unit of land yields a prot of $7 if devoted to wheat, of $3 if devoted to potatoes.
Should the summer be rainy, a unit of land yields a prot of $1 if devoted to wheat, of
$3 if devoted to potatoes. Good and bad weather is equally likely. Farmer is risk averse

and has 40 units of land. Let the farmer's utility function be given by u(x) = 4 x,
where x stays for wealth.
(a) (1 p.) What is the optimal allocation of land between potatoes and wheat.
Illustrate graphically and explain the result intuitively.
Solution
Suppose p is amount of land devoted to potatoes and 40-p to wheat.
p p p p
Eu = 21 4 3p + 7(40 p)+ 21 4 3p + (40 p) = 2 3p + 7(40 p)+2 3p + (40 p).
d(Eu) 2 2
Lets solve maximization task
d(p)
= x+20 70x = 0 => x+20
2
= 70x
4
=> p =
d(Eu)

10 => 40 p = 30 and it is maximum because d2 (p) (10, 30) < 0. Thus Eu = 12 15
and optimal allocation is 10 potatoes and 30 wheat.
Intuitively it is clear because in summer weather farmer gets much more from wheat
than from potatoes while if weather is rainy the dierence in favor of potatoes is not
so big. Also, because farmer is risk-averse he prefer to allocate some potato even if it is
decrease expected income.
In order to build the graph lets rst denote w=40-p and manipulate with expected
utility equation.
Eu4 64Eu2 w+576w2
Eu = 2 3p + 7w + 2 3p + w => p = 48Eu2

(b) (1.5 p.) Suppose that farmer is oered to rent his land out for one period for X
rubles. Find the smallest X at which the farmer would accept the oer. Denote this sum
by Xmin and illustrate graphically. Compare Xmin with the one period expected prot
from his business and explain the result.
Solution u2
In (a) it was shown that Eu

= 12 15. Lets rst nd reverse of utility function x = 16
.
(12 15)2
Now it is clear Xmin =
16
= 135$. While expected prot is Ep= 21 (3 10 + 7(40
1
10)) + 2 (3 10 + (40 10)) = 150$. Xmin < Ep because expected prot undermines
uncertainty unwanted for this risk-averse farmer. So 135$ for certain is better than 150$

30
in average (under given preferences structure and probabilities) because it may result in
240$ as well as in 60$ and it scare risk-averse farmer.
For butiful graphical illustration lets nd to which combination of p and w on our

indierence curve (Eu = 12 15) corresponds 135$ expected prot. In order to do it
lets make some trick and solve the system

(12 15)4 64(12 15)2 w+576w2
( (
p=
48(12 15)2 p = 45
1 1
=>
2
(3p + 7w) + 2 (3p + w) = 150 w=0

(c) (1.5 p.) Now, suppose that the farmer can purchase an insurance against bad
weather that pays $1 per unit of insurance in case of bad weather at a price of $0.5
per unit. Find the optimal allocation of land and the quantity of insurance purchased.
Illustrate graphically and explain the result intuitively.
Solution
It is absolutely unclear which weather is good and which is bad. However lets assume
that rain is bad weather. Also suppose that
p p I is amount of insurance bought.
Eu = 2 3p + 7(40 p) 0.5I + 2 3p + (40 p) + 0.5I p
pLets solve maximization expected utility problem : max(2 3p + 7(40 p) 0.5I +
2 (3p + (40 p) + 0.5I) on I 0, 0 p 40
2
70p0.125I =0
2

2p+40+0.5I
=0
The structure of this system of equation says that no stationar points can be derived so
maximization problem will not have interior solution. Lets check for corner solutions. It

is clear from (a) that if I=0 than Eu = 12 15. But if p=0 than I=240 and Eu = 16 10
that is bigger! It is also global maximum because no stationar points exists and another
optimal allocation of land is 40 wheats
corner solution less than this one. Thus
and quantity of insurance purchased is 240$.
Intuitively farmer will get many money from wheats if weather is good and many
money from insurance if weather is bad (benets rather much in both cases each time
thanks to wheats xor insurance) thus he prefers such allocation strategy.
It is not clear what exactly important to show on graphic. For example interesting
to set I variable but 3D graphics are prohibited. However lets draw indeerence curves

with dierent I values and Eu = 16 10.

31
The main conslusion possible to make from this graph is that when I (insurance)
increase M RSwheats,potatoes also in average growth up. In other words increase in insurance
associated with rise of wheat atractiveness compare with potatoes. Intuitibely its happens
because potatoes itself played some insurance role for bad times before and than it
substittes by insurance.

4.1 Third homework



First task (2.5 p.) Production function of the rm is given by Q = 4 L+ K. The
wage of labor is $8 per hour and the rental price of capital is $2 per hour.
(a) Derive short run AC and AVC. Explain the shapes of AVC and AC.
Solution
In short run suppose that capital is xed . So denote it K = Kf where f
6
undermines
xed.
p
Now lets solve cost minimization task: minimize 8L + 2Kf on q = 4 L + Kf

q 2 +Kf 2 Kf q
From constraint we have L = . Inserting it into costs equation we get
16
q 2 +5Kf 2 Kf q
C(q) = 2
and it is minimum because for any q only one possible value
L 0 may correspond while capital is xed.
5Kf
q 2 +5Kf 2
q we get:
C(q) Kf q q+ q
p
SRAC = q
= 2q
= 2
Kf and taking part including

q 2 +5Kf 2 Kf q
q
p
SRAV C = q
2
= 2
Kf
d(SRAC) 5K
The shapes of SRAC and SRAVC are
d(q)
= 12 2q2f , d(SRAV
d(q)
C)
= 12 . Here we see
that while production is low average costs will grow slowly or even decrease. But when
production will be hight enoughft costs will start to increase. It is due to decreasing
5Kf
M PL = 2L => d(M PL )
d(L)
= 1 1
2 3 < 0. Asymptotically shape is limq> 2 2q 2 = 2
L
1

(b) Derive long run total costs and nd long run average cost. Are there economies
of scale? Explain the result.
Solution
First lets solve cost minimization task: minimize
8L + 2K on q =4 L+ K
L = 8L + 2K (4 L + K q)

6 Result will be approximately similar if Labour instead of Capital is xed

32
2

8 L = 0
( = 4 L = 4 K
(
q2
=4 L=4 K L=K=
2 2K = 0 => => L = K => 25

4 L+ K q =0
= 4q
5
4 L+ K =0 5 L=q
q2 2
q
Analyzing Hessian at point ( 25 , 25 , 4q ) lets check whether it is minimum:
5 10 5

0
2L 1

0 2K 10 4q
q 2q

0 = det q ( q2 ) 32 0 = ( 2q5 )2 4q5 ( 10q )2 4q5 < 0


5

2
det L 3

L2 2 3 2 3
( q25 ) 2 4( q25 ) 2
25

4q
21K 0
5
4K 2
3 2q 0 5
q2 3
4( 25 ) 2
Thus we have found minimum. Finnaly substituting this result in total costs function
2q 2
we get LRT C = .
5
LRAC = q = 2q
LRT C
5
2q
LRAV C = 5

is noeconomies on scale because [tQ(L, K) = t(4 L + K)] > [Q(tL, tK) =
Here
t(4 L + K)], t > 1. So it is no surprise that SRAC and LRAVC increase with
amount of producion.
(c) Illustrate on the same graph LRAC and several SRAC curves.
Solution

Second task (1.5 p.)A constant-cost industry consists of a large number of rms,
each
( of which has a cost function of the form:
2
9q + 16, q > 0
0, q = 0
(a) Find the long run supply curve of an individual rm;
Solution
d(9q 2 +16) p
MC = d(q)
= 18q => p = 18q => LRSi = 18
where LRSi denotes Long-Run
supply of individual rm . i

33
(b)
Find the long run equilibrium price and the number of operating rms, if the
d
market demand is given by Q (p) = 150 4p.
Solution
Here rst it is important to nd which price is minimal accepted by rms. So when
pq < 9q 2 + 16 => p < 9q + 16q
rms will leave the market and when p 9q +
16
q
they
will not enter because no positive prots could be achieved. Under competitive market
assumption which undermines zero prots and using individual suply function we get
p
p 9 18 + 16
p => p = 24 that is
18
Long-Run competitive price
.
p
Equilibrium condition is N
18
= 150 4p and using LR competitive price we get
N 24
18
= 150 4 24 => N = 40.5 that is number of operating rms
7
.

Third task (2.5 p.) Consider a perfectly competitive industry that produces good
X. All rms in this industry have identical technologies with cost function c(q), where
c'(q)>0, c(q)>0 for q>0. Unfortunately a fraction of the output produced by each
rm is defective and cannot be sold. Moreover rm experiences some utilization cost for
unsold output and the corresponding cost function is given by l(z), where z is the volume
of utilized output, l'(0)=0, l'(z)>0 and l(z)>0 for z>0. Both production and utilization
cost are zero if output is zero.
(a) Suppose that improvement in management brings a reduction of while cost
function stays the same. What is the impact on individual supply of each rm? Provide
both graphical and analytical solution (Note: change in is not necessarily small).
Solution
d(c+l(q))
Total costs of rm including utilization are c+l(q) from this we get M C =
d(q)
=
0
c + l0 (q). Because prot maximization condition requires p=MC and d(M C)
d()
= 0 +

7 Here is one interesting thought. If the number of rms must be integer it will be 40 because the last
rm will sell only part of what it whants and thus will have negative prots so it will prevent this
rm from enterance or force to leave. Also it will cause an increase in prices and thus in prots of
other rms. Thus prots of rms in constant-cost industry with integer number of rms rise with
fractional part of N and even particular function can be constructed. But all of this go beyong of
the scope of this task.

34
d(l(q)) d(l(q)) d(l(q))
d(q)
+ 2 d(q)d() where from l0 (z) > 0 we get
d(q)
> 0 and from l00 (z) > 0 it is
8 d(l(q)) d(M C) 9
evidient that
d(q)d()
> 0 thus
d()
> 0 consuquently any reduction in decrease

MC thus increasing equilibrium level of output . This analiticaly prove that any
10

reduction in increase equilibrium level of output and vice versa.

(b) Suppose that there are N rms in the industry. Denote by p() the equilibrium
price in this market. Find the impact of reduction in on this short-run equilibrium
price. Explain the result. (No need in graphical solution).
Solution
From (a) we get
d(M C)
d()
> 0 and because p() = M C we get d(p())
d()
> 0 consequently
any reduction in will decrease short-run equilibrium price. It is because
decrease in decrease utilization costs it leads to decrease in marginal costs and it
results in decrease of price in conditions of perfect competitive market.
To be even more convicing lets remeber that prot maximization condition requires
p=MC and here we have prot maximization task (1 )p q c l and consequently
d(p) 0 (1)+M C
prot maximization condition (1)p = M C => p =
MC
(1)
=> d() = M C (1) 2 >0

Fourth task (2 p.) Consider perfectly competitive constant cost industry with identical
rms. Suppose that, a per-unit sales subsidy is replaced by a lump sum subsidy that every
active rm gets. The lump-sum subsidy leave the equilibrium price the same as it was
under the per-unit subsidy scheme. Compare the total government expenditures under
per unit and lump-sum subsidy schemes: (i) graphically assuming U-shaped AC and (ii)
analytically (i.e. for any type of AC consistent with the conditions of the problem).
Solution
ii) Suppose ls denotes lump-sum while pu undermines per unit. Initial zero prot
pq c(q) = 0 and for per-unit subsidy we have (p + s)q c(q) = 0. Thus if
condition is
p0 then subsidy price will be p = p0 s and q0 = qpu . Zero prot condition
initial price is
LS
for Lump-Sum is p q +
Q
q c(q) = 0 and could be rewriten as (p0 s) q + LS
Q
q c(q) =
8 Because we can interprete increase in q and as an increase in z
9 d(M C) > 0 satises for any point thus result is correct for any, even large changes.
d()
10 Dueto the fact that increase (decrease) in MC leads to rise (fall) in p (because of prot maximization
condition MC=p) and increase (decrease) in prices results in decrease (increase) in demand value
thus reducing (increasing) equilibrium level of output.

35
(p0 s + LS
Q
) q c(q) = 0 nally we get p0 s + LS Q
= c(q)
q
= AC(q). Because
M Cpu = M Cls s => M Cpu < M Cls => qpu > qls => q0 > qls for any xed price

including p . We know that (p0 s +
LS
Q
) = AC(qls ) and (p0 s) = p = AC(qpu ) =
AC(q0 ) => (p0 s + LS Q
) > AC(q0 ) = p0 => LSQ
> s => LS > sQ consequently
Lump-Sum subsidy is greater then per unit.

Fifth task (2,5 p.) Consider the rental housing market with linear demand and supply
curves. Suppose that house owners are required to pay rental tax equal to share (0 <
< 1) of rental price paid by tenants. All these tax revenue goes to the local government
budget.
(a) Local government decided to introduce rent control policy. According to this
regulation the rental price for tenants cannot exceed Pcontrol . Assume that Pcontrol corresponds
to the market-clearing price in absence of rental tax. Illustrate the initial equilibrium and
new equilibrium at the same graph in terms of price paid by tenants. Compare initial
(with tax but before rent control) and new (with tax and rent control) values of CS, PS,
local government surplus (GS) and TS by lling in the following table. Comment on the
welfare impact of the policy.
Solution
Prot maximization under tax is (1)pq c. Solving (1)pq c and substituting it
d
in Q after corresponding solution we obtain ptax which is equilibrium (market-clearing)
price under tax. Repeating the same for without tax pq c we get pcontrol . So it is clear
that ptax > pcontrol because taxes undermine additional costs which will be compensated
by higher price level and control price equals to without tax price. Thus this market will
defenitely face reduction in production after price control. The most interesting is that
any supply price with tax will be (1 ) higher then price without tax. I will think that
11
government surplus is amount of taxes collected by government .

11 While I also could but will not think GS as an area between Stax and Swithout tax .

36
Initial New Change
R q0 R D(q) R q1 R D(q)
CS d(p)d(q) d(p)d(q) uncertain because ptax > pcontrol but q0 > q1
R q00 R pptax
tax R 0q1 R ppcontrol
PS 0 Stax (q)
d(p)d(q) 0
control
Stax (q)
d(p)d(q) decrease because ptax > pcontrol and q0 > q1 Because TS
GS ptax q0 pcontrol q1 decrease because ptax > pcontrol and q0 > q1
TS decrease by qq10 0D(q) d(p)d(q)
R R
CSinitial + P Sinitial CSnew + P Snew
decrease welfare also decrease thus impact of policy in total is negative. But because CS
maybe increase policy maybe increased (as well as maybe decreased) consumer surplus
thus consumer maybe better.
(b) What was the implicit assumption about who would get the apartment under
price control? How the value of loss would change if this assumption is altered (illustrate
by graph the maximum possible additional loss).
Solution
Under assumption the most rich of those who accept control price will get these
appartments. If not they (alter of assumption) it will result in decrease of consumer
surplus because dierence between willing to pay and control price will be less. In the
worst case ( maximum additional loss) the most poor part of the society which accept
control price will buy apartments instead of the most rich. In general if assumption is
altered loss will increase.

37
5 Fourth homework
First task (1 p.) Question is based on Figure 11.5 `Third degree price discrimination'
(p.407 P&R). This gure represents graphical derivation of equilibrium under third-
degree price discrimination when monopolist has increasing marginal cost. Read the
following comment to Figure.11.5: the total quantity produced, QT = Q1 + Q2 , is found
by summing the marginal revenue curves M R1 and M R2 horizontally, which yields the
dashed curve M RT , and nding its intersection with the marginal cost curve. As we
know the rst order conditions suggest that marginal revenue for each group separately,
not the total marginal revenue should be equal to marginal cost. Is there a contradiction?
What equation do you solve by constructing this M RT ? Explain your answers.
Solution
As we can see from the graph M RT (Q1 + Q2 ) = M R1 (Q1 ) = M R2 (Q2 ) = M C(Q1 +
Q2 ) thus here is no contradiction. Principle of constructing M RT is following. For every
point Q we take M R1 (Q). Then we look for a such M R2 that M R2 (Z) = M R1 (Q). Then
we say that M RT (Z + Q) = M R1 (Q). Using this approach for all Q we nnaly get M RT
curve!

Second task (4 p.) A monopolist has two customers with the following demand
functions: Q1 = 10 p and Q2 = 12 p. The monopolist's cost function is T C(Q) =
0.25Q2 .
(a) Suppose that monopolist can dierentiate between the customers and oers two
part taris. Find the optimal two part taris. Provide both analytical and graphical
solutions.

38
Solution12 R Qi R 10Q Q2i
Because consumer surplus is CSi =
0 10Qi
dpdQ =
2
we solve trivial task
Q21 Q22 2 Q2
max[ 2 + 2 + (10 Q1 )Q1 + (12 Q2 )Q2 0.25(Q1 + Q2 ) ] where Ti = CSi = 2i and
9
nally we get Q1 = , Q2 =
2
13
2
consequently usage taris are
p1 = 11
2
, p2 = 11
2
. Entry
92 13 2
taris are T1 = CS1 = 2
2
= 10.125, T2 = CS2 = 2
2
= 21.125.

(b) Now suppose that monopolist cannot discriminate between the groups. Find the
optimal two part tari.
Solution
Surplus will be less for the rst consumer thus it will be used for T. Solving trivial
(10p)2
prot maximization task max[
2
+ (10 p)p + (12 p)p 0.25(10 p + 12 p)2 ]
we get usage tarip = 6 while Q1 = 4, Q2 = 6. Entry tari
is T =
(106)2
2
= 8.
(c) Calculate the value of society loss in (a) and (b), Compare and explain the result.
Solution (
22 2p, if p 10
First lets nd perfect society TS. Aggregate demand function is Q =
12 p, if p > 10
Q Q
thus in optimal point M C(Q) = P (Q) => 11
2
= 2 => Q = 11 consequently
R 11 R 11 Q2 R 2 R 12Q
TS=
2 Q d(p)d(Q) + 0 Q d(p)d(Q) = 61.5
2 2
13
RQ
Denoting T Sa , T Sb total surplus in (a) and (b) and using fomula T S = p(Q)d(Q)
0
T C(Q) we have:
T Sa = RT S = 61.5 thus loss
R 6 are zero.
T Sb = 0 (10Q)d(Q)+ 0 (12Q)d(Q)0.25(4+6)2 = 61 thus loss are 61.5-61=0.5
4

12 More eaasy approach to solve this task is to think about maximum possible total surplus and set
p=MC. It is because all surplus might be achieved by monopolist as a prot by using dicrimination.
13 Because else we will each time calculate MC from zero which is incorrect

39
There is no loss in (a) because monopolist due to discrimination could transfer surplus
in prot thus prot maximization for him is similar to surplus maximization. Abscence of
discrimination between groups in (b) results in more losses because surplus maximization
in this case is not a way of prot maximization because ones group surplus will be
achieved by monopolist only partially.

Third task (6 p.) In town N only one rm oers job to engineers. The inverse labour
F F F
supply curve by female engineers is given by L (w ) = max(0, w 4) and inverse
M M M k
labour supply curve of male engineers is L (w ) = w , where w stays for wage rate
of group k (k = M, F ).
Assume that labour is the only variable factor in the short run
2
and the short run production function of the rm is F (L) = max(0, 13L 0.5L ). The
nal product is sold at perfectly competitive market and the price is $4 per unit.
(a) Find the prot maximizing wage rates assuming that the rm can set dierent
wage rates to male and female engineers. Illustrate by graph
Solution
We solve trivial task max[(13(wF 4 + wM ) 0.5(wF 4 + wM )2 ) 4 wF (wF
4) wM (wM )] and get prot maximizing wage rates: wM = 6, wF = 8. Thus rm
2
output will be 13 (6 4 + 8) 0.5 (6 4 + 8) = 80. The most strange moment here is
that prot is not zero while the (
market is perfectly competitive. Consequently demand

, p 4
function on this market is Qd =
0, p > 4
It is dicult to make graphical illustration because wages are direnet for each sex.
For example demand for female labour depends on male wage. Thus I will provide male
labour demand as a function of female wage and vice versa. Solving prot maximization
task treating one sex wage as a constant and substituting the result in labour supply
M 342wF F 242wM
function we get labour demand functions LD = and LD = .
3 3

40
(b) Now assume that price discrimination is not allowed any more. Find equilibrium.
Illustrate graphically on the same graph with dierent colour.
Solution
We solve trivial task max[(13(w 4 + w) 0.5(w 4 + w)2 ) 4 w(w 4) w2 ] and
get prot maximizing wage rate: w = 7. Thus rm output will be 13 (7 4 + 7)
0.5 (7 4 + 7)2 = 80.
A little bit another, less algebraical more economical approach to solution is
M P L = d(F p)
d(L)
= 52 4L
L = w 4 + w => L = 2w 4 => w = L2 + 2
d(L( L +2))
M F C = d(Lw)
d(L)
= 2
d(L)
= L + 2 (when w>4 )
M F C = M P L => L + 2 = 52 4L => 5L = 50 => L = 10 => w = 10 2
+2 =7
I am do not understand what on the same graph means and dierent from what
should be used colors.

If the task is to draw new information on graph (a) illustration will be following

41
(c) Compare social welfare in (a) and (b) by calculating the dierence in the total
surplus? Provide intuitive explanation (explain carefully).
Solution
I will not take take into account consumers surplus on market where commodity is
sold. I will operationalizae total surplus in this case as a sum of rm prot and integral
dierence between what people was willing to work for and which wage they achieve as
a result.
R6R6 R 84 R 8
T Sa = 0 L dwdL + 0 dwdL + 80 4 6 6 8 (8 4) = 278
R7R7 R 74 R L+4
7
T Sb = 0 L dwdL + 0 L+4
dwdL + 80 4 7 7 7 (7 4) = 279
Social welfare higher in (b) because of abscenc of sexual discrimiantion in wages. While
rm's prots are the same in both cases workers surplus is higher in (b) indicating that
absence of dicrimination benets workers in total.
(d) Suggest taxes or subsidies which can be used to induce the discriminating monopsony
from part (a) to choose ecient employment. Demonstrate that proposed solutions would
result in ecient outcome.
Solution
I am absolutely did not understand what is ecient employment. I search over internet
very deep and did not nd any useful information thus this term is missleading. I will
operationalize it as maximum TS (see my denition of TS for this case in (c)) that could
be achieved on this market.
M 2 F F
First lets solve surplus maximization taskmax[ (w 2 ) + w (w2 4) + (13(wF 4 + wM )
0.5(w 4+w ) )4w (w 4)w (w )] and get wM = 20
F M 2 F F M M
3
, wF = 26
3
and substituting
this in maximized function we get T S = 290.
F M F
If bubsidy is not part of TS, then solving max[(13(w 4 + w ) 0.5(w 4 +

wM )2 ) (4 + s) wF (wF 4) wM (wM )] we get wM = 15(s+4)


2(s+5)
, wF = 19s+80
2s+10
. Solving

42
(
15(s+4)
2(s+5)
= 20
3
we get s = 4. Thus susbidy 4 will guarantee the result.
19s+80 26
2s+10
= 3

5.1 Fifth homework

First task (5 p.) Two companies, A and B, operate in the same gas eld. Each
company's costs depend on its own production level as well as production level of the
2 2 2 2
other company: CA = 0.25(qA + qB ) + 0.5qA and CA = 0.25(qA + qB ) + 0.5qB . Demand
for gas is given by Q(p) = 20 p.
(a) Suppose that the two rms are oligopolists that compete by choosing production
levels simultaneously and independently. Find equilibrium.
Solution:
Here we deal with Cournout model. Suppose qB is xed then prot maximization task
for A will be (20 qA qB ) qA 0.25(qA + qB )2 0.5qA2 consequently response function
403qB
of A is qA = 7
and because both rms costs function are symmetric to each other
403qA
response function of B is qB = . Substituting one function into another we get
7
403qA
qA =
403(
7
7
)
=> qA = 4 => qB = 4 that is Cournot equilibrium
(b)
How would your answer to part (a) change if the two rms acted as price takers
at gas market? Find equilibrium.
Solution:
It is not clear whether the price is already given or the task is about Bertrand model
or abour perfect competitive market.
I. If the task is about Bertrand model
0.25(qA +qB )2 +0.5qA
2 0.25(qA +qB )2 +0.5qA
2 0.25(qA +qB )2 +0.5qA
2
ACA = qA
, ACB = qB
; p = ACA = ACB => qA
=
0.25(qA +qB )2 +0.5qA
2

qB
=> qA = qB => p = 1.5qA => qA = 2p 3
=> qA + qB = 4p 3
=> 4p 3
=
60
20 p => p = 7 consequently Bertrand equilibrium is qA = qB =
40
7
so the result
has changed.
II. If the task is about given price
2 2
Here we solve the task p A 0.25(qA + qB ) 0.5qA
q( getting response functions
(
2pqB 2pqA
3 3
qA = , qB = thus Nash equilibrium requires qA =
0 if qB > 2p 0 if qA > 2p
2pqA
2p
3
3
=> qA = p2 => qB = p2
If p=10 the answer will be qA = q B = 5.
If p < 10 then Nash equilibrium condition
p
(not values) is still q A = pB =
2
. But if p>10 then rms will met the problem of
overproduction and equilibrium will be any levels of production and no Nash equilibrium
could be found without additional assumptions about sells of each rm under overproduction.
So the result has changed.
III. If we treat price takers as perfect competition
1
Perfect competition requries p=MC thus p = 0.5(qA +qB )+qA => qA = (2pqB ), p =
3
0.5(qA +qB )+qB => qB = 13 (2pqA ); q = qA +qB = 13 (2pqB )+ 31 (2pqA ) = 43 p 13 q =>
4
3
q = 43 p => q = p substituting in market equation 20p = p => p = 10 => q1 = q2 = 5

43
(c) Calculate and compare the values of deadweight loss in (a) and (b). Explain the
result.
Solution:
(20p)2
Solving maximization task
2
+ p(20 p) 0.5(20 p)2 ( 20p
2
)2 we get p=12
and TS=80.
R 4+4 R 20Q
T Sa = 0 2044
dQdp + (20 4 4) 4 0.25(4 + 4)2 0.5 42 + (20 4 4)
4 0.25(4 + 4) 0.5 42 = 80
2
2
T SbII = p2 + p2 0.5p2 ( p2 )2 = 0.75p2 particulary if p=10 then T SbIII (10) = 75
2
DWL loss in (a) are zero while in (b) are 80 0.75p and if p=10 then 80 75 = 5
DWL in (b) are greater because price taking undermines restriction on natural price
formation. It is even more evidient if we think that under price 12 in (b) we will have
overproduction while in (a) we will maximize TS.
(d) Is it possible to eliminate the loss from ineciency (if there is any) found in (c)
for case (a) and /or case (b) via taxes/subsidies? Find the required taxes/subsidies or
prove that it is impossible.
Solution:
No subsidy need in (a) because cournout equilibrium gives maximum possible TS in
this particular case.(
We know that maximum TS
(requires qA = qB = 4 and p=12 thus we
p+s
qA = qB = 2 = 4 p+s= 8
need in (b) to have => => s = 4. Consequently
p = 12 p = 12
introducing tax equals to 4 will give ecient outcome in (b).

44
Second task Suppose rm A has two dierent technologies of producing output that
could be used in any combination. Technology 1 requires four units of labor and one
unit of raw material to produce each unit of output. Technology 2 requires two units of
labor and one unit of raw material per unit of output but before technology 2 can be
used rm has to incur setup costs equal to 70 and there are no setup costs for technology
1. Suppose that wage rate is 3 and the price of raw materials is 4. The inverse demand
function is P (Q) = 34 Q
(a) If rm A is the only rm operating in this industry, which technology should it
use. Find corresponding output, price and prot.
Solution:
I am absolutely confused about any combination. But because all functions are
linear it is not sucient how to understand this remark. But the task could be much
more interesting and challenging in case of nonlinear cost functions.
First we solve minimization costs task for both technologies in order to achieve
minimize 3l + 4k on q = min(k, 0.25l) results in C1 = 16q
minimize 3l + 4k + 70 on q = min(k, 0.5l) results in C2 = 10q + 70
Solving prot maximization task under rst technologi (34 q)q 16q we get 1 = 81
while solving prot maximization task under second technologi (34 q)q 10q 70 we
get 2 = 74 thus 1 > 2 consequently the rst technologi is better.
(b) Suppose that rm A is afraid that company B is going to enter the market. Firm
B's cost function is given by T C B (q) = 10q and it has to purchase a license that cost 90
had it decided to enter the industry. If B enters the market then the rms will compete
by choosing quantities simultaneously. Which technology would you advise rm A to
choose (if this choice is made before rm B decides, whether to enter or not)? Represent
the game tree, nd equilibrium and explain the result.
Solution: ( (
12 q2B , if qB 24 9 q2B , if qB 18
Response functions are qA2 = , qA1 =
0, if qA > 24 0, if qB > 18
(
12 q2A , if qA 24
qB = .
0, if qA > 24
Lets now nd Nash equilibriums (substituting one into another corresponding reaction
functions) for the following situations
1. Firm B enter and A use rst technologie
qA = 4, qB = 10 => A = 16, B = 10
2. Firm B enter and A use second technologie
qA = 8, qB = 8 => A = 6, B = 26
3. Firm B do not enter and A use rst technologie
A = 81, B = 0
4. Firm B do not enter and A use second technologie
A = 74, B = 0
First rm must choose second technologie to prevent rm B from entering
because in this case rm B will have negative prot.

45
Using Cermela-Kun algorithm we nd SPNE in (2 technology, (enter, not enter)).
(c) Find ecient allocation. Calculate and compare the value of DWL in (a) and (b).
What can you conclude about the role of potential competition? Explain the result.
Solution:
qa2
To get maximum possible TS (ecient allocation) we solve maximization task
2
+
(34 qa )qa 10qa 70 and get TS=218.
2
T Sa = 92 + (34 9) 9 10 9 = 175.5 => DW La = 112.5
2
T Sb = 122 + (34 12) 12 10 12 70 = 146 => DW Lb = 144
(d) Explain intuitively the relevance of sunk costs to the ability of the incumbent (rm
A) in part (b) to make a commitment to use dierent technology. Demonstrate, how the
game tree and the resulting outcome would dier when the set-up cost associated with
the second technology are not sunk.
Solution:
If the second's technologie costs are sunk rm B can expect that if even rm A will
choose the second technologie, after rm B enter the market rm A will switch it's
technologie to the rst (in order to increase prots). Consequently rm B can enter
asserting that rm A will choose rst technologie after B enter thus making prots for B
positive. Thus incumbent rm will have less power to force rm B not enter the market:
corresponding commitment will be hardly achieved.

46
The resulting outcome will be SPNE ((1 technology, 1 technology, 1 technology),
(enter, enter)). So rm A will choose 1 technology while rm B will enter the market.

47

You might also like